correctd mistakes partII
authorhackbard <hackbard>
Fri, 23 May 2003 12:34:49 +0000 (12:34 +0000)
committerhackbard <hackbard>
Fri, 23 May 2003 12:34:49 +0000 (12:34 +0000)
ising/ising.tex

index 14cf570..6916d92 100644 (file)
@@ -36,7 +36,8 @@ Sie ist eine fundamentale Gr"o"se in der statistischen Physik. Von ihr k"onnen v
 \end{itemize}
 
 \section{Phasen"uberg"ange}
 \end{itemize}
 
 \section{Phasen"uberg"ange}
-Die Phase ist eine m"ogliche Zustandsform eines makroskopischen Systems im thermischen Gleichgewicht. Unterschiedliche Phasen "au"sern sich in unterschiedlichen Werten makroskopischer Observablen. Beispiele:
+Die Phase ist eine m"ogliche Zustandsform eines makroskopischen Systems im thermischen Gleichgewicht. Unterschiedliche Phasen "au"sern sich in unterschiedlichen Werten makroskopischer Observablen.\\
+Beispiele:
 \begin{itemize}
 \item Dichte ($H_2O$)
 \item Magnetisierung (Nickel)
 \begin{itemize}
 \item Dichte ($H_2O$)
 \item Magnetisierung (Nickel)
@@ -81,10 +82,10 @@ Dann lautet die Hamilton-Funktion:
 \]
 Man erahnt: ($J > 0$, \, Ferromagnet)
 \begin{itemize}
 \]
 Man erahnt: ($J > 0$, \, Ferromagnet)
 \begin{itemize}
-\item $T \to 0$: \\ $\longrightarrow \textrm{Zustand niedrieger Energie} \longrightarrow \textrm{Spins gleich ausgerichtet}$ \\ $\longrightarrow \textrm{hohe Magnetisierung}$
-\item $T \to \infty$: \\ $\longrightarrow \textrm{Zustand hoher Energie} \longrightarrow \textrm{Spins zufaellig ausgerichtet}$ \\ $\longrightarrow \textrm{keine Magnetisierung}$
+\item $T \to 0$: \\ $\longrightarrow \textrm{Zustand niedriger Energie} \longrightarrow \textrm{Spins gleich ausgerichtet}$ \\ $\longrightarrow \textrm{hohe Magnetisierung}$
+\item $T \to \infty$: \\ $\longrightarrow \textrm{Zustand hoher Energie} \longrightarrow \textrm{Spins zuf"allig ausgerichtet}$ \\ $\longrightarrow \textrm{keine Magnetisierung}$
 \end{itemize}
 \end{itemize}
-Unter einer bestimmten Temperatur stellt sich auch ohne Aenderung eines aeusseren Magnetfeldes eine spontane Magnetisierung ein.\\
+Unter einer bestimmten Temperatur stellt sich auch ohne "Anderung eines "au"seren Magnetfeldes eine spontane Magnetisierung ein.\\
 \\
 Molekularfeldn"aherung:\\
 Approximation des Ising Modells durch Vernachl"assigung der Spinfluktuationen $S_i-<S_i>$. Damit kann man den Spin-Wechselwirkungs-Term umschreiben:
 \\
 Molekularfeldn"aherung:\\
 Approximation des Ising Modells durch Vernachl"assigung der Spinfluktuationen $S_i-<S_i>$. Damit kann man den Spin-Wechselwirkungs-Term umschreiben:
@@ -138,7 +139,7 @@ Man findet also einen Phasen"ubergang unabh"angig von der Gitterdimension. Die f
 
 \chapter{Loesungen des Ising Modells}
 
 
 \chapter{Loesungen des Ising Modells}
 
-\section{1-dimensionale L"osung}
+\section{L"osung f"ur $d=1$}
 \setlength{\unitlength}{1.5cm}
 \begin{picture}(10,1)
  \thicklines
 \setlength{\unitlength}{1.5cm}
 \begin{picture}(10,1)
  \thicklines
@@ -159,10 +160,14 @@ Die Hamilton-Funktion in einer Dimension lautet:
 \[
  H = - \sum_{i=1}^{N} J_{i,i+1} S_i S_{i+1} - \mu B_0 \sum_{i=1}^{N} S_i
 \]
 \[
  H = - \sum_{i=1}^{N} J_{i,i+1} S_i S_{i+1} - \mu B_0 \sum_{i=1}^{N} S_i
 \]
-Annahme: periodische Randbedingungen, $S_{N+1} \equiv S_1$ \, (Translationsinvarianz), $J_{i,i+1} \equiv J$ \\
+Annahmen:
+\begin{itemize}
+ \item periodische Randbedingungen, $S_{N+1} \equiv S_1$
+ \item Translationsinvarianz, $J_{i,i+1} \equiv J$
+\end{itemize}
 Abkuerzung: $K = \beta J$, $h = \mu B \beta$ \\
 \\
 Abkuerzung: $K = \beta J$, $h = \mu B \beta$ \\
 \\
-Die Energie des Systems ist nun gegebn durch:
+Die Energie des Systems ist nun gegeben durch:
 \[
  E = -J \sum_{i=1}^{N} S_i S_{i+1} - \mu B_0 \sum_{i=1}^{N} S_i
 \]
 \[
  E = -J \sum_{i=1}^{N} S_i S_{i+1} - \mu B_0 \sum_{i=1}^{N} S_i
 \]
@@ -182,14 +187,14 @@ Finde Matrix $\mathbf{T}$ mit fogenden Eigenschaften:
  <S_i|\mathbf{T}|S_{i,i+1}> = e^{ K S_i S_{i,i+1} + \frac{h}{2} ( S_i + S_{i,i+1} )} \\
  \\
  \textrm{also:} \\
  <S_i|\mathbf{T}|S_{i,i+1}> = e^{ K S_i S_{i,i+1} + \frac{h}{2} ( S_i + S_{i,i+1} )} \\
  \\
  \textrm{also:} \\
- <1|\mathbf{T}|1> = e^{K+h} \\
- <-1|\mathbf{T}|-1> = e^{K-h} \\
- <1|\mathbf{T}|-1> = <-1|\mathbf{T}|1> = e^{-K} \\
+ \displaystyle <1|\mathbf{T}|1> = e^{K+h} \\[2mm]
+ \displaystyle <-1|\mathbf{T}|-1> = e^{K-h} \\[2mm]
+ \displaystyle <1|\mathbf{T}|-1> = <-1|\mathbf{T}|1> = e^{-K} \\[2mm]
  \\
  wobei: \\
  \begin{array}{ll}
  \\
  wobei: \\
  \begin{array}{ll}
-  |S_i = +1> = \left( \begin{array}{c} 1 \\ 0 \end{array} \right) \, \textrm{,} &
-  |S_i = -1> = \left( \begin{array}{c} 0 \\ 1 \end{array} \right)
+  \displaystyle |S_i = +1> = \left( \begin{array}{c} 1 \\ 0 \end{array} \right) \, \textrm{,} &
+  \displaystyle |S_i = -1> = \left( \begin{array}{c} 0 \\ 1 \end{array} \right)
  \end{array}
 \end{array}
 \]
  \end{array}
 \end{array}
 \]
@@ -207,12 +212,12 @@ Die Matrix muss also wie folgt aussehen:
 Damit laesst sich die Zustandssumme neu schreiben:
 \[
  \begin{array}{ll}
 Damit laesst sich die Zustandssumme neu schreiben:
 \[
  \begin{array}{ll}
- Z & = \sum_{S_1} \sum_{S_2} \ldots \sum_{S_N} <S_1|\mathbf{T}|S_2> <S_2|\mathbf{T}|S_3> \ldots <S_{N-1}|\mathbf{T}|S_N> <S_N|\mathbf{T}|S_1> \\
-   & = \sum_{S_1} <S_1|\mathbf{T}^N|S_1> \\
-   & = \textrm{Sp} \, \mathbf{T}^N
+ \displaystyle Z & = \sum_{S_1} \sum_{S_2} \ldots \sum_{S_N} <S_1|\mathbf{T}|S_2> <S_2|\mathbf{T}|S_3> \ldots <S_{N-1}|\mathbf{T}|S_N> <S_N|\mathbf{T}|S_1> \\[2mm]
+ \displaystyle  & = \sum_{S_1} <S_1|\mathbf{T}^N|S_1> \\[2mm]
\displaystyle  & = \textrm{Sp} \, \mathbf{T}^N
  \end{array}
 \]
  \end{array}
 \]
-Wegen der Vollstaendigkeit der Spinzustaende ist $\mathbf{T}$ diagonalisierbar und die Spur Darstellungsunabhaengig. Aus $\textrm{det} ( \mathbf{T} - \lambda \mathbf{1} ) = 0$, erhaelt man folgende Eigenwerte:
+Wegen der Vollstaendigkeit der Spinzustaende kann obere Vereinfachung vorgenommen werden. $\mathbf{T}$ ist diagonalisierbar und die Spur Darstellungsunabhaengig. Aus $\textrm{det} ( \mathbf{T} - \lambda \mathbf{1} ) = 0$, erhaelt man folgende Eigenwerte:
 \[
  \lambda_{\pm} = e^{K} ( \cosh h \pm \sqrt{\sinh^2 h + e^{-4K}} )
 \]
 \[
  \lambda_{\pm} = e^{K} ( \cosh h \pm \sqrt{\sinh^2 h + e^{-4K}} )
 \]
@@ -223,19 +228,19 @@ Daraus folgt:
 Fuer den Fall $B_0 = 0$ gilt:
 \[
  \begin{array}{l}
 Fuer den Fall $B_0 = 0$ gilt:
 \[
  \begin{array}{l}
-  \lambda_{\pm} = e^K \pm e^{-K} \\
-  Z = 2^N \cosh^N (K) + 2^N \sinh^N (K) = 2^N \cosh^N (K) (1 + \tanh^N (K)) \stackrel{N >> 1}{\longrightarrow} 2^N \cosh^N (K) \\
-  F = -k_B T \, \textrm{ln} \, Z \stackrel{N >> 1}{\longrightarrow} -N k_B T \, \textrm{ln} \, (2 \cosh (\beta J))
+  \displaystyle \lambda_{\pm} = e^K \pm e^{-K} \\[2mm]
+  \displaystyle Z = 2^N \cosh^N (K) + 2^N \sinh^N (K) = 2^N \cosh^N (K) (1 + \tanh^N (K)) \stackrel{N >> 1}{\longrightarrow} 2^N \cosh^N (K) \\[2mm]
+  \displaystyle F = -k_B T \, \textrm{ln} \, Z \stackrel{N >> 1}{\longrightarrow} -N k_B T \, \textrm{ln} \, (2 \cosh (\beta J))
  \end{array}
 \]
 Dabei wurde verwendet, dass $\lambda_+^N$ im thermodynamischen Limes viel groesser ist als $\lambda_-^N$. \\
 Fuer die Magnetisierung mit Magnetfeld gilt:
 \[
  \begin{array}{ll}
  \end{array}
 \]
 Dabei wurde verwendet, dass $\lambda_+^N$ im thermodynamischen Limes viel groesser ist als $\lambda_-^N$. \\
 Fuer die Magnetisierung mit Magnetfeld gilt:
 \[
  \begin{array}{ll}
-  M & = \frac{1}{Z} \sum_{\{S\}} (\sum_{i} \mu S_i) e^{-\beta H} \\
-    & = \frac{1}{\beta} (\frac{\partial}{\partial{B_0}} \, \textrm{ln} \, Z) \\
-    & \stackrel{N >> 1}{\longrightarrow} \frac{N}{\beta \lambda_+} \frac{\partial{\lambda_+}}{\partial{B_0}} \\
-    & = N \mu \frac{\sinh (\beta \mu B_0)}{\sqrt{\cosh^2 (\beta \mu B_0) - 2e^{-2 \beta J} \sinh (2 \beta J)}}
+  \displaystyle M & = \frac{1}{Z} \sum_{\{S\}} (\sum_{i} \mu S_i) e^{-\beta H} \\[2mm]
+  \displaystyle & = \frac{1}{\beta} (\frac{\partial}{\partial{B_0}} \, \textrm{ln} \, Z) \\[2mm]
+  \displaystyle & \stackrel{N >> 1}{\longrightarrow} \frac{N}{\beta \lambda_+} \frac{\partial{\lambda_+}}{\partial{B_0}} \\[2mm]
+  \displaystyle & \displaystyle = N \mu \frac{\sinh (\beta \mu B_0)}{\sqrt{\cosh^2 (\beta \mu B_0) - 2e^{-2 \beta J} \sinh (2 \beta J)}}
   
  \end{array}
 \]
   
  \end{array}
 \]
@@ -258,16 +263,16 @@ Erkenntnis:\\
 \item magnetisches Moment verschwindet fuer alle endlichen Temperaturen wenn $B_0 = 0$
 \item es gibt keinen Phasenuebergang fuer das eindimensionale Isingmodell fur $T>0$
 \end{itemize}
 \item magnetisches Moment verschwindet fuer alle endlichen Temperaturen wenn $B_0 = 0$
 \item es gibt keinen Phasenuebergang fuer das eindimensionale Isingmodell fur $T>0$
 \end{itemize}
-F"ur $T=0$ kann obere Approximation nichtmehr verwendet werden, da gilt:
+F"ur $T=0$ kann obere Approximation nicht mehr verwendet werden, da gilt:
 \[ 
  \lim_{T \rightarrow 0} \frac{\lambda_+}{\lambda_-}=1
 \]
 \[ 
  \lim_{T \rightarrow 0} \frac{\lambda_+}{\lambda_-}=1
 \]
-Mann kann zeigen, da"s bei $B_0=T=0$ ein Phasen"ubergagng liegt (Korrelationsl"ange geht gegen unendlich), und eine spontane Magnetisierung existiert. Kritische Exponenten:
+Mann kann zeigen, da"s bei $B_0=T=0$ ein Phasen"ubergang liegt (Korrelationsl"ange geht gegen unendlich), und eine spontane Magnetisierung existiert. Kritische Exponenten:
 \[
  \alpha = 1 \qquad \beta = 0 \qquad \gamma = 1
 \]
 
 \[
  \alpha = 1 \qquad \beta = 0 \qquad \gamma = 1
 \]
 
-\section{2-dimensionale L"osung}
+\section{L"osung f"ur $d=2$}
 W"ahrend das eindimensionale Modell noch relativ leicht zu l"osen war, und deshalb hier detailliert beschrieben wurde, ist das zweidimensionale h"ochst nichttrivial. Es wird auf eine genau L"osung verzichtet. Tats"achlich ist das einzige Problem die Diagonalisierung einer $2^N \times 2^N$ - Matrix (wieder Transfer-Matrix-Methode). Eine L"osung wird nur ohne vorhandenes Magnetfeld gefunden.\\
 \\
 Hamiltonian:
 W"ahrend das eindimensionale Modell noch relativ leicht zu l"osen war, und deshalb hier detailliert beschrieben wurde, ist das zweidimensionale h"ochst nichttrivial. Es wird auf eine genau L"osung verzichtet. Tats"achlich ist das einzige Problem die Diagonalisierung einer $2^N \times 2^N$ - Matrix (wieder Transfer-Matrix-Methode). Eine L"osung wird nur ohne vorhandenes Magnetfeld gefunden.\\
 \\
 Hamiltonian:
@@ -281,26 +286,26 @@ Dabei geben die Indizes der Spins deren Punkt im Gitter an. Dies schreiben wir k
 wobei
 \[
 \begin{array}{ll}
 wobei
 \[
 \begin{array}{ll}
- E(\mu_j,\mu_k) & \equiv - \sum_{i=1}^N S_{i,j} S_{i,k} \\
- E(\mu_j)       & \equiv - J \sum_{i=1}^N S_{i,j} S_{i+1,j} - \mu B_0 \sum_{i,j} S_j \\
- \mu_j          & \equiv \{S_{1,j},\ldots,S_{N,j}\}
+ \displaystyle E(\mu_j,\mu_k) & \displaystyle \equiv - \sum_{i=1}^N S_{i,j} S_{i,k} \\[2mm]
+ \displaystyle E(\mu_j)       & \displaystyle \equiv - J \sum_{i=1}^N S_{i,j} S_{i+1,j} - \mu B_0 \sum_{i,j} S_j \\[2mm]
+ \displaystyle \mu_j          & \displaystyle \equiv \{S_{1,j},\ldots,S_{N,j}\}
 \end{array}
 \]
 Damit bestimmen wir analog zum eindimensionalen Fall eine Transfer-Matrix $\mathbf{T}$, mit Matrixelementen:
 \[
 \end{array}
 \]
 Damit bestimmen wir analog zum eindimensionalen Fall eine Transfer-Matrix $\mathbf{T}$, mit Matrixelementen:
 \[
- <\mu_j|\mathbf{T}|\mu_k> = e^{- \beta \Big( E(mu_j,\mu_k) + E(\mu_j) \Big)}
+ <\mu_j|\mathbf{T}|\mu_k> = e^{- \beta \Big( E(\mu_j,\mu_k) + E(\mu_j) \Big)}
 \]
 Dies ist eine $2^N \times 2^N$ - Matrix, die es wie erw"ahnt zu diagonalisieren gilt. Analog zum $d=1$ Fall gilt:
 \[
 \]
 Dies ist eine $2^N \times 2^N$ - Matrix, die es wie erw"ahnt zu diagonalisieren gilt. Analog zum $d=1$ Fall gilt:
 \[
- Z = \textrm{Sp} \, mathbf{T}^N
+ Z = \textrm{Sp} \, \mathbf{T}^N
 \]
 \]
-Diesen Schritt kann man sich zum Beispiel in [\ref{lit7}] genauer anschauen. Im Folgenden Werden nur die Endresultate betrachtet.\\
+Diesen Schritt kann man sich zum Beispiel in [\ref{lit7}] genauer anschauen. Im Folgenden werden nur die Endresultate betrachtet.\\
 \\
 F"ur die freie Energie pro Spin $f = \lim_{N \to \infty} \frac{1}{N} (-k_B T \, \textrm{ln} \, Z)$ erh"alt man
 \[
  f = -k_B T \textrm{ln} \, \Big( 2 \cosh(2 \beta J) \Big) - \frac{k_B T}{2 \pi} \int_{0}^{\pi} d\phi \, \textrm{ln} \, \frac{1}{2} \Bigg( 1 + \sqrt{1 - K^2 \sin^2 \phi} \Bigg)
 \]
 \\
 F"ur die freie Energie pro Spin $f = \lim_{N \to \infty} \frac{1}{N} (-k_B T \, \textrm{ln} \, Z)$ erh"alt man
 \[
  f = -k_B T \textrm{ln} \, \Big( 2 \cosh(2 \beta J) \Big) - \frac{k_B T}{2 \pi} \int_{0}^{\pi} d\phi \, \textrm{ln} \, \frac{1}{2} \Bigg( 1 + \sqrt{1 - K^2 \sin^2 \phi} \Bigg)
 \]
-mit $K = \frac{2}{\cosh (2 \beta J) \coth (2 \beta J)}$, und demnach f"ur die Magnetisierung:
+mit $\displaystyle K = \frac{2}{\cosh (2 \beta J) \coth (2 \beta J)}$, und demnach f"ur die Magnetisierung:
 \[
  m = \left\{
  \begin{array}{ll}
 \[
  m = \left\{
  \begin{array}{ll}
@@ -324,7 +329,7 @@ Fazit:
 \item auch ohne vorhandenes Magnetfeld hat der Ising Ferromagnet eine spontane Magnetisierung wenn $T < T_C$
 \end{itemize}
 
 \item auch ohne vorhandenes Magnetfeld hat der Ising Ferromagnet eine spontane Magnetisierung wenn $T < T_C$
 \end{itemize}
 
-\section{3-dimensionale L"osung}
+\section{L"osung f"ur $d=3$}
 Das dreidimensionale Modell kann bis heute nicht exakt analytisch gel"ost werden. Approximationen und Monte Carlo Simulationen liefern jedoch ueberzeugende Resultate. Man erwartet von der analytisch exakten L"osung keine weiteren Informationen mehr.\\
 \\
 Das dreidimensionale Ising Modell zeigt Phasenueberg"ange.
 Das dreidimensionale Modell kann bis heute nicht exakt analytisch gel"ost werden. Approximationen und Monte Carlo Simulationen liefern jedoch ueberzeugende Resultate. Man erwartet von der analytisch exakten L"osung keine weiteren Informationen mehr.\\
 \\
 Das dreidimensionale Ising Modell zeigt Phasenueberg"ange.
@@ -335,19 +340,19 @@ Im Folgenden soll gezeigt werden wie man durch sogenannte Monte Carlo Simulation
 Gesucht sei der Erwartungswert $<A>$.
 \[
 \begin{array}{l}
 Gesucht sei der Erwartungswert $<A>$.
 \[
 \begin{array}{l}
- <A> = \sum_i p_i A_i \, \textrm{, wobei} \\
- p_i = \frac{e^{- \beta E_i}}{\sum_j e^{\beta E_j}} \, \textrm{Boltzmann Wahrscheinlichkeitsverteilung} \\
- E_i \, \textrm{Energie im Zustand i}
+ \displaystyle <A> = \sum_i p_i A_i \, \textrm{, wobei} \\[2mm]
+ \displaystyle p_i = \frac{e^{- \beta E_i}}{\sum_j e^{\beta E_j}} \, \textrm{Boltzmann Wahrscheinlichkeitsverteilung} \\[2mm]
\displaystyle E_i \, \textrm{Energie im Zustand i}
 \end{array}
 \]
 \end{array}
 \]
-Anstatt ueber alle Zust"ande zu summieren, greift man nur einige zuf"allige Zust"ande auf, deren Wahrscheinlichkeit idealerweise nat"urlich der Boltzmannverteilung entspricht.
+Anstatt ueber alle Zust"ande zu summieren, greift man nur einige zuf"allige Zust"ande auf, deren Wahrscheinlichkeit idealerweise nat"urlich der Boltzmannverteilung entspricht (importance sampling).
 \[
  <A>_{est} = \frac{1}{N} \sum_{i=1}^{N} A(i)
 \]
 \[
  <A>_{est} = \frac{1}{N} \sum_{i=1}^{N} A(i)
 \]
-$N$ entspricht hierbei der Anzahl der Itterationen in der Computersimulation.
+$N$ entspricht hierbei der Anzahl der Iterationen in der Computersimulation.
 \begin{itemize}
  \item $P(A,t)$ sei die Wahrscheinlichkeit der Konfiguration $A$ zur Zeit $t$
 \begin{itemize}
  \item $P(A,t)$ sei die Wahrscheinlichkeit der Konfiguration $A$ zur Zeit $t$
- \item $W(A \rightarrow B)$ sei Wahrscheinlichkeit pro Zeiteinheit, da"s dir Konfiguration von $A$ nach $B$ wechselt
+ \item $W(A \rightarrow B)$ sei Wahrscheinlichkeit pro Zeiteinheit, da"s die Konfiguration von $A$ nach $B$ wechselt
 \end{itemize}
 Damit gilt:
 \[
 \end{itemize}
 Damit gilt:
 \[
@@ -356,7 +361,7 @@ Damit gilt:
 und f"ur gro"se $t$ ist dir willk"urliche Anfangskonfiguration vergessen, $P(A,t) \rightarrow p(A)$.\\
 Eine Bedingung f"ur eine zeitunabh"angige Wahrscheinlichkeitsverteilung ist:
 \[
 und f"ur gro"se $t$ ist dir willk"urliche Anfangskonfiguration vergessen, $P(A,t) \rightarrow p(A)$.\\
 Eine Bedingung f"ur eine zeitunabh"angige Wahrscheinlichkeitsverteilung ist:
 \[
- W(A \rightarrow B) P(A,t) = W(B \rightarrow A) P(B,t)
+ W(A \rightarrow B) P(A,t) = W(B \rightarrow A) P(B,t) \qquad \textrm{(detailed balance)}
 \]
 und somit gilt:
 \[
 \]
 und somit gilt:
 \[
@@ -375,7 +380,7 @@ Der Pseudocode eines Programms k"onnte nun wie folgt aussehen:
 \item Gehe alle Gitterplaetze durch
 \item Berechne $\delta E$ fuer Spinflip (Naechste Nachbarn anschauen)
 \item Wenn $\delta E < 0$ flip, ansonsten nur wenn Zufallszahl kleiner $e^{\frac{-\delta E}{k_B T}}$
 \item Gehe alle Gitterplaetze durch
 \item Berechne $\delta E$ fuer Spinflip (Naechste Nachbarn anschauen)
 \item Wenn $\delta E < 0$ flip, ansonsten nur wenn Zufallszahl kleiner $e^{\frac{-\delta E}{k_B T}}$
-\item Spins aufsummieren, dies entspricht der Magnetisierung (nach genuegend vielen Itterationen ($N^3$))
+\item Spins aufsummieren, dies entspricht der Magnetisierung (nach genuegend vielen Iterationen ($N^3$))
 \end{itemize}
 
 \chapter{Anwendungen}
 \end{itemize}
 
 \chapter{Anwendungen}
@@ -395,12 +400,13 @@ Der Pseudocode eines Programms k"onnte nun wie folgt aussehen:
   \item Hamilton: $H = - \sum J_{ij} S_i S_j - \mu B_0 \sum S_i$, wobei die $J_{ij}$ zufaellige, symmetrisch um $0$ verteilte Kopplung darstellt
   \end{itemize}
  \end{itemize}
   \item Hamilton: $H = - \sum J_{ij} S_i S_j - \mu B_0 \sum S_i$, wobei die $J_{ij}$ zufaellige, symmetrisch um $0$ verteilte Kopplung darstellt
   \end{itemize}
  \end{itemize}
+\newpage
 \item Spingl"aser: Optimierung und Ged"achtnis [\ref{lit8}]
  \begin{itemize}
   \item Traveling Salesman Problem:
    \begin{itemize}
 \item Spingl"aser: Optimierung und Ged"achtnis [\ref{lit8}]
  \begin{itemize}
   \item Traveling Salesman Problem:
    \begin{itemize}
-   \item "Aufheizen" des Systems, Wegstrecken bekommen gleiche Gewichtung
-   \item "Abk"uhlen des Systems, Zustand niedrigster Energie stellt sich ein, der ideale Weg?
+   \item \dq Aufheizen \dq des Systems, Wegstrecken bekommen gleiche Gewichtung
+   \item \dq Abk"uhlen \dq des Systems, Zustand niedrigster Energie stellt sich ein, der ideale Weg?
    \end{itemize}
   \item Ged"achtnis:
   \begin{itemize}
    \end{itemize}
   \item Ged"achtnis:
   \begin{itemize}